K
Khách

Hãy nhập câu hỏi của bạn vào đây, nếu là tài khoản VIP, bạn sẽ được ưu tiên trả lời.

29 tháng 8 2015

A= 1/3 + 1/9 + 1/27 + 1/81 + 1/243

Ax3=(1/3 + 1/9 + 1/27 + 1/81 + 1/243)x3

Ax3=1/3 x 3 + 1/9 x 3 + 1/27 x 3  + 1/81 x 3 + 1/243 x 3

Ax3=1+1/3+1/9+1/27+1/81

Ax3-A=(1+1/3+1/9+1/27+1/81)-(1/3+1/9+1/27+1/81+1/243)

Ax(3-1)=1-1/243

Ax2=243/243-1/243

Ax2=      242/243

A   = 242/243:2

A    =  242/243 x 1/2

A    =       121/243

Vậy A= 121/243

Li-ke cho mik nhé!

Ta có:\(A=\frac{1}{3}+\frac{1}{9}+\frac{1}{27}+\frac{1}{81}+\frac{1}{243}\)

Xét\(\frac{1}{3}A=\frac{1}{9}+\frac{1}{27}+\frac{1}{81}+\frac{1}{243}+\frac{1}{729}\)

\(\Leftrightarrow A-\frac{1}{3}A=\frac{1}{3}-\frac{1}{729}\)

\(\Leftrightarrow\frac{2}{3}A=\frac{243-1}{729}\Leftrightarrow A=\frac{3}{2}\times\frac{242}{729}=\frac{121}{243}\)

31 tháng 8 2019

Phải là : A=1/3+1/9+1/27+1/81+1/243 ta có: 3A=1+1/3+1/9+1/27+1/81                     3A-A=(1+1/3+1/9+1/27+1/81)-(1/3+1/9+1/27+1/81+1/243)=1-1/243       2A=242/243                                             A=242/243:2=121/243

11 tháng 9 2016

đặt S=\(\frac{1}{3}+\frac{1}{9}+\frac{1}{27}+\frac{1}{81}+\frac{1}{243}+\frac{1}{729}\)

=>3S= \(1+\frac{1}{3}+\frac{1}{9}+\frac{1}{27}+\frac{1}{81}+\frac{1}{243}\)

=>3S-S=\(\left(1+\frac{1}{3}+...+\frac{1}{243}\right)-\left(\frac{1}{3}+\frac{1}{9}+...+\frac{1}{729}\right)\)

=>s=1-1/729 = 728/729

11 tháng 9 2016

1/3+1/9+1/27+1/81+1/243+1/729=(1/3+1/9+1/81)+(1/27+1/243+1/729)=37/81+37/729=333/729+37/729=370/729

26 tháng 2 2020

\(\frac{1}{3}+\frac{1}{9}+\frac{1}{27}+\frac{1}{81}+\frac{1}{243}\)

\(=\frac{3}{9}+\frac{1}{9}+\frac{1}{27}+\frac{1}{81}+\frac{1}{243}\)

\(=\frac{4}{9}+\frac{1}{27}+\frac{1}{81}+\frac{1}{243}\)

\(=\frac{12}{27}+\frac{1}{27}+\frac{1}{81}+\frac{1}{243}=\frac{13}{27}+\frac{1}{81}+\frac{1}{243}=\frac{39}{81}+\frac{1}{81}+\frac{1}{243}=\frac{40}{81}+\frac{1}{243}\)

\(=\frac{120}{243}+\frac{1}{243}=\frac{121}{243}\)

23 tháng 4 2016

1 + 1/3 + 1/9 + 1/27 + 1/81 + 1/243 + 1/729

=1+ 243/729+ 81/729 + 27/729 + 9/729 + 3/729

=1093/729

23 tháng 4 2016

1 + 1/3 + 1/9 + 1/27 + 1/81 + 1/243 + 1/729

=1093/729

16 tháng 5 2015

1+ 1 /3+1/9+1/27+1/81+1/243+1/729.
Đặt:
S = 1 + 1/3 + 1/9 + 1/27 + 1/81 + 1/243 
Nhân S với 3 ta có:
S x 3 = 3 +1+ 1/3 + 1/9 + 1/27 + 1/81
Vậy: 
S x 3 - S = 3 - 1/243
2S = 728/243
S = 364/243

tick đúng nha

4 tháng 8 2017

=364/243

13 tháng 9 2018

Đặt \(A=\frac{1}{3}+\frac{1}{9}+\frac{1}{27}+\frac{1}{81}+\frac{1}{243}\) ta có : 

\(A=\frac{1}{3}+\frac{1}{3^2}+\frac{1}{3^3}+\frac{1}{3^4}+\frac{1}{3^5}\)

\(3A=1+\frac{1}{3}+\frac{1}{3^2}+\frac{1}{3^3}+\frac{1}{3^4}\)

\(3A-A=\left(1+\frac{1}{3}+\frac{1}{3^2}+\frac{1}{3^3}+\frac{1}{3^4}\right)-\left(\frac{1}{3}+\frac{1}{3^2}+\frac{1}{3^3}+\frac{1}{3^4}+\frac{1}{3^5}\right)\)

\(2A=1-\frac{1}{3^5}\)

\(A=\frac{1-\frac{1}{3^5}}{2}\)

Vậy \(A=\frac{1-\frac{1}{3^5}}{2}\)

Chúc bạn học tốt ~ 

6 tháng 11 2016

https://www.youtube.com/watch?v=fvGaHwKrbUc

6 tháng 11 2016

Đặt A = 1/3 + 1/9 + 1/27 + 1/81 + 1/243 + 1/729 
A x 3 = 3 x (1/3 + 1/9 + 1/27 + 1/81 + 1/243 + 1/729) 
= 1 + 1/3 + 1/9 + 1/27 + 1/81 + 1/243 
A x 3 - A = 1 + 1/3 + 1/9 + 1/27 + 1/81 + 1/243 - (1/3 + 1/9 + 1/27 + 1/81 + 1/243 + 1/729) 
= 1 + 1/3 + 1/9 + 1/27 + 1/81 + 1/243 - 1/3 - 1/9 - 1/27 - 1/81 - 1/243 - 1/729 
= 1 - 1/729 
A x 2 = 728/729 
A = 364/729

2 tháng 8 2015

đặt biểu thức trên là A

ta có : 

A= ghi biểu thức ra

A.3=3.(1+1/3+1/9+1/27+1/81+1/243+1/729)

A.3=3+1+1/3+1/9+1/27+1/81+1/243

A.3-A=...

A.2=3-1/729

sau đó bn tự tính ra

9 tháng 6 2023

\(A=1+\dfrac{1}{3}+\dfrac{1}{9}+\dfrac{1}{27}+\dfrac{1}{81}+\dfrac{1}{243}+\dfrac{1}{729}\)

\(3A=3+1+\dfrac{1}{3}+\dfrac{1}{9}+\dfrac{1}{27}+\dfrac{1}{81}+\dfrac{1}{243}\)

\(3A-A=\left(3+1+\dfrac{1}{3}+\dfrac{1}{9}+\dfrac{1}{27}+\dfrac{1}{81}+\dfrac{1}{243}\right)-\left(1+\dfrac{1}{3}+\dfrac{1}{9}+\dfrac{1}{27}+\dfrac{1}{81}+\dfrac{1}{243}+\dfrac{1}{729}\right)\)

\(2A=3-\dfrac{1}{729}=\dfrac{2186}{729}\)

\(A=\dfrac{2186}{729}\div2=\dfrac{1093}{729}\)

9 tháng 6 2023

A = \(1+\dfrac{1}{3}+\dfrac{1}{9}+\dfrac{1}{27}+\dfrac{1}{81}+\dfrac{1}{243}+\dfrac{1}{729}\)

3A = \(3+1+\dfrac{1}{3}+\dfrac{1}{9}+\dfrac{1}{27}+\dfrac{1}{81}+\dfrac{1}{243}\)

3A - A = ( \(3+1+\dfrac{1}{3}+\dfrac{1}{9}+\dfrac{1}{27}+\dfrac{1}{81}+\dfrac{1}{243}\) ) - ( \(1+\dfrac{1}{3}+\dfrac{1}{9}+\dfrac{1}{27}+\dfrac{1}{81}+\dfrac{1}{243}+\dfrac{1}{729}\) )

2A = 3 - \(\dfrac{1}{729}=\dfrac{728}{729}\)

A = \(\dfrac{728}{729}:2=\dfrac{364}{729}\)